LSAT and Law School Admissions Forum

Get expert LSAT preparation and law school admissions advice from PowerScore Test Preparation.

User avatar
 Dave Killoran
PowerScore Staff
  • PowerScore Staff
  • Posts: 5852
  • Joined: Mar 25, 2011
|
#26555
Complete Question Explanation
(The complete setup for this game can be found here: lsat/viewtopic.php?t=11091)

The correct answer choice is (A)

Answer choice (A) is the correct answer choice.

Answer choice (B) can be eliminated because P’s performance must be after M’s performance.

Answer choice (C) can be eliminated because P’s performance must be second or fifth.

Answer choice (D) can be eliminated because J’s performance must be immediately after K’s performance.

Answer choice (E) can be eliminated because when L performs third, N must perform fifth.

Get the most out of your LSAT Prep Plus subscription.

Analyze and track your performance with our Testing and Analytics Package.